You are on page 1of 20

CONSTITUTIONAL LAW II : ARREST SEARCHES AND SEIZURES

Valmonte vs. De Villa searches and seizures is a personal right invocable subversive movements for the maintenance of peace
only by those whose rights have been infringed, or in the state. The aerial target zoning were intended
threatened to be infringed. Not all searches and to flush out subversives and criminal elements
Facts: On 20 January 1987, the National Capital seizures are prohibited. Those which are reasonable coddled by the communities were the said drives
Region District Command (NCRDC) was activated are not forbidden. The setting up of the questioned were conducted. They said that they have
pursuant to Letter of Instruction 02/87 of the checkpoints may be considered as a security intelligently and carefully planned months ahead for
Philippine General Headquarters, AFP, with the measure to enable the NCRDC to pursue its mission the actual operation and that local and foreign media
of establishing effective territorial defense and joined the operation to witness and record such
mission of conducting security operations within its
maintaining peace and order for the benefit of the event.
area of responsibility and peripheral areas, for the public. Checkpoints may not also be regarded as
purpose of establishing an effective territorial measures to thwart plots to destabilize the govt, in
defense, maintaining peace and order, and providing the interest of public security. Between the inherent Issue: Whether or Not the saturation drive
an atmosphere conducive to the social, economic right of the state to protect its existence and committed consisted of violation of human rights.
promote public welfare and an individuals right
and political development of the National Capital
against a warrantless search w/c is, however,
Region. As part of its duty to maintain peace and reasonably conducted, the former should prevail. Held: It is not the police action per se which should
order, the NCRDC installed checkpoints in various True, the manning of checkpoints by the military is be prohibited rather it is the procedure used or the
parts of Valenzuela, Metro Manila. Petitioners aver susceptible of abuse by the military in the same methods which "offend even hardened
that, because of the installation of said checkpoints, manner that all governmental power is susceptible of sensibilities" .Based on the facts stated by the
abuse. But, at the cost of occasional inconvenience, parties, it appears to have been no impediment to
the residents of Valenzuela are worried of being
discomfort and even irritation to the citizen, the securing search warrants or warrants of arrest before
harassed and of their safety being placed at the checkpoints during these abnormal times, when any houses were searched or individuals roused from
arbitrary, capricious and whimsical disposition of the conducted w/in reasonable limits, are part of the sleep were arrested. There is no showing that the
military manning the checkpoints, considering that price we pay for an orderly society and a peaceful objectives sought to be attained by the "aerial
their cars and vehicles are being subjected to regular community. zoning" could not be achieved even as th rights of
the squatters and low income families are fully
searches and check-ups, especially at night or at
protected. However, the remedy should not be
dawn, without the benefit of a search warrant and/or GUANZON VS. DE VILLA [181 SCRA 623; G.R.
brought by a tazpaer suit where not one victim
court order. Their alleged fear for their safety 80508; 30 JAN 1990]
complaints and not one violator is properly charged.
increased when, at dawn of 9 July 1988, Benjamin In the circumstances of this taxpayers' suit, there is
Parpon, a supply officer of the Municipality of Facts: The 41 petitioners alleged that the "saturation no erring soldier or policeman whom the court can
drive" or "aerial target zoning" that were conducted order prosecuted. In the absence of clear facts no
Valenzuela, Bulacan, was gunned down allegedly in
in their place (Tondo Manila) were unconstitutional. permanent relief can be given.
cold blood by the members of the NCRDC manning They alleged that there is no specific target house to
the checkpoint along McArthur Highway at Malinta, be search and that there is no search warrant or In the meantime where there is showing that some
Valenzuela, for ignoring and/or refusing to submit warrant of arrest served. Most of the policemen are abuses were committed, the court temporary
himself to the checkpoint and for continuing to speed in their civilian clothes and without nameplates or restraint the alleged violations which are shocking to
identification cards. The residents were rudely rouse the senses. Petition is remanded to the RTC of
off inspire of warning shots fired in the air.
from their sleep by banging on the walls and Manila.
windows of their houses. The residents were at the
Issue: WON the installation of checkpoints violates point of high-powered guns and herded like cows.
AURELIO S. ALVERO vs ARSENIO P. DIZON, et
the right of the people against unreasonable Men were ordered to strip down to their briefs for the
al.,
police to examine their tattoo marks. The residents
searches and seizures
complained that they're homes were ransacked,
tossing their belongings and destroying their
Held: Petitioner's concern for their safety and valuables. Some of their money and valuables had
disappeared after the operation. The residents also G.R. No. L-342 May 4, 1946
apprehension at being harassed by the military
manning the checkpoints are not sufficient grounds reported incidents of maulings, spot-beatings and
to declare the checkpoints per se, illegal. No proof maltreatment. Those who were detained also
has been presented before the Court to show that, in suffered mental and physical torture to extract
the course of their routine checks, the military, confessions and tactical informations. The FACTS: The petitioner has been accused of treason;
indeed, committed specific violations of petitioners'' respondents said that such accusations were all lies. that at the hearing on his petition for bail, the
rights against unlawful search and seizure of other Respondents contends that the Constitution grants to
rights. The constitutional right against unreasonable government the power to seek and cripple
1|Pa g e
Copyright2015-2018 T.P. All rights reserved.
CONSTITUTIONAL LAW II : ARREST SEARCHES AND SEIZURES

prosecution presented, as part of its evidence, as the means by which it was committed. operating procedures. Upon opening the package, he
certain documents which had been allegedly seized noticed a suspicious odor which made him took
by soldiers of the United States Army, accompanied sample of the substance he found inside. He reported
by Filipino Guerrillas in the petitioners house. The this to the NBI and invited agents to his office to
Petitioner further contends that the seized The Petitioner consented to the presentation of the inspect the package. In the presence of the NBI
documents should be returned as it obtained by seized documents, as part of the evidence for the agents, Job Reyes opened the suspicious package
means of force and intimidation or through coercion, prosecution, at the hearing in his petition for bail and and found dried-marijuana leaves inside. A case was
those are not his personal papers but part of the files at the trial of the case on the merits, without having filed against Andre Marti in violation of R.A. 6425 and
of the New Leaders Association, which was proven to insisted that the question of the alleged illegality of was found guilty by the court a quo. Andre filed an
be an organization created for the purpose of the search and seizure of said papers and documents appeal in the Supreme Court claiming that his
collaborating with the enemy. Lastly, the should first have been directly litigated and constitutional right of privacy was violated and that
presentation of the seized documents in the trial is established by a motion. the evidence acquired from his package was
tantamount to compelling him to testify against inadmissible as evidence against him.
himself, in violation of his constitutional rights.
Issue: Can the Constitutional Right of Privacy be
COMPULSORY SELF-INCRIMINATION enforced against private individuals?

ISSUES: Whether or not the seized documents are


legal? Ruling: The Supreme Court held based on the
speech of Commissioner Bernas that the Bill of Rights
Not violated by the use of evidence of articles governs the relationship between the individual and
obtained by an unconstitutional search and seizure. the state.
Thus, the petitioner is estopped from questioning
Whether or not the documents seized should be their admission.
admitted as evidence in the trial court? The constitutional proscription against unlawful
searches and seizures therefore applies as a restraint
directed only against the government and its
agencies tasked with the enforcement of the law. It is
PURPOSE: (Adam vs New York)
HELD. No. The petition for Certiorari with Injunction not meant to be invoked against acts of private
is absolutely no merit. individuals. It will be recalled that Mr Job Reyes was
the one who opened the box in the presence of the
NBI agents in his place of business. The mere
The purpose of the constitutional provisions against presence of the NBI agents did not convert the
unlawful searched and seizures is to prevent reasonable search effected by Mr. Reyes into a
RATIONALE:The right of the officer and men of the violations of private security in person and property, warrantless search and siezure proscribed by the
United States Army to arrest the petitioner as a and unlawful invasions of the sanctity of the home, constitution. Merely to observe and look at that
collaborationist suspect, and to seize his personal by officers of the law acting under legislative and which is in plain sight is not a search.
papers is unquestionable. Also, proclamation of judicial sanction, and to give remedy against such
General Douglas McArthur, as Commander in Chief of usurpations when attempted.
the United States of Army, declaring his purpose to The judgement of conviction finding appeallant guilty
remove certain citizens of the Philippines, who had beyond reasonable doubt of the crime charged was
voluntarily given aid and comfort to the enemy, in AFFIRMED.
violation of the allegiance.
PEOPLE OF THE PHILIPPINES vs. ANDRE MARTI
(193 SCRA 57) [G.R. No.L-32409. February 27, 1971.]

BACHE & CO. (PHIL.), INC. and FREDERICK E.


EXCEPTION: Important exception to the necessity Facts: On August 14, 1987, the appellant and his SEGGERMAN, Petitioners, v. HON. JUDGE
for a Search Warrant is the right of search and common-law wife, Shirley Reyes went to Manila VIVENCIO M. RUIZ, MISAEL P. VERA, in his
seizure as an incident to a lawful arrest. A lawful Packaging and Export Forwarders to send packages capacity as Commissioner of Internal Revenue,
arrest may be made either while a crime is being to Zurich, Switzerland. It was received by Anita Reyes ARTURO LOGRONIO, RODOLFO DE LEON,
committed or after its commission. The right to and ask if she could inspect the packages. Shirley GAVINO VELASQUEZ, MIMIR DELLOSA, NICANOR
search includes in both instances that of searching refused and eventually convinced Anita to seal the ALCORDO, JOHN DOE, JOHN DOE, JOHN DOE,
the person of him who is arrested, in order to find package making it ready for shipment. Before being and JOHN DOE, Respondents.
and seize things arrested with the crime as its fruits sent out for delivery, Job Reyes, husband of Anita and
proprietor of the courier company, conducted an San Juan, Africa, Gonzales & San Agustin, for
inspection of the package as part of standard Petitioners.
2|Pa g e
Copyright2015-2018 T.P. All rights reserved.
CONSTITUTIONAL LAW II : ARREST SEARCHES AND SEIZURES

the warrant. The agents nevertheless proceeded with were not even taken. At this juncture it may be well
Solicitor General Felix Q. Antonio, Assistant their search which yielded six boxes of documents. to recall the salient facts. The transcript of
Solicitor General Crispin V .Bautista, Solicitor stenographic notes (pp. 61-76, April 1, 1970, Annex J-
Pedro A. Ramirez and Special Attorney Jaime On March 3, 1970, petitioners filed a petition with the 2 of the Petition) taken at the hearing of this case in
M. Maza for Respondents. Court of First Instance of Rizal praying that the the court below shows that per instruction of
search warrant be quashed, dissolved or recalled, respondent Judge, Mr.Eleodoro V. Gonzales, Special
that preliminary prohibitory and mandatory writs of Deputy Clerk of Court, took the depositions of the
FACTS: Respondent Misael P. Vera, Commissioner of
injunction be issued, that the search warrant be complainant and his witness, and that stenographic
Internal Revenue, wrote a letter addressed to
declared null and void, and that the respondents be notes thereof were taken by Mrs. Gaspar. At that time
respondent Judge Vivencio M. Ruiz requesting the
ordered to pay petitioners, jointly and severally, respondent Judge was at the sala hearing a case.
issuance of a search warrant against petitioners for
damages and attorneys fees. On March 18, 1970, After respondent Judge was through with the hearing,
violation of Section 46(a) of the National Internal
the respondents, thru the Solicitor General, filed an Deputy Clerk Gonzales, stenographer Gaspar,
Revenue Code, in relation to all other pertinent
answer to the petition. After hearing, the court, complainant De Leon and witness Logronio went to
provisions thereof, particularly Sections 53, 72, 73,
presided over by respondent Judge, issued on July 29, respondent Judges chamber and informed the Judge
208 and 209, and authorizing Revenue Examiner
1970, an order dismissing the petition for dissolution that they had finished the depositions. Respondent
Rodolfo de Leon, one of herein respondents, to make
of the search warrant. In the meantime, or on April Judge then requested the stenographer to read to
and file the application for search warrant which was
16, 1970, the Bureau of Internal Revenue made tax him her stenographic notes. Special Deputy Clerk
attached to the letter.
assessments on petitioner corporation in the total Gonzales testified it.
sum of P2,594,729.97, partly, if not entirely, based
In the afternoon of the following day, February 25,
on the documents thus seized.
1970, respondent De Leon and his witness, The search warrant does not particularly describe the
respondent Arturo Logronio, went to the Court of First things to be seized.
Instance of Rizal. They brought with them the Issue: WON the search warrant is valid
following papers: respondent Veras aforesaid letter- The documents, papers and effects sought to be
request; an application for search warrant already seized are described in Search Warrant No. 2-M-70 in
Held: No. The elements of valid search warrant are
filled up but still unsigned by respondent De Leon; an this manner:jgc:chanrobles.com.ph
not present in this case, the requisites are: 1. It must
affidavit of respondent Logronio subscribed before
be issued upon probable cause;
respondent De Leon; a deposition in printed form of "Unregistered and private books of accounts
respondent Logronio already accomplished and (ledgers, journals, columnars, receipts and
2. The probable cause must be determined
signed by him but not yet subscribed; and a search disbursements books, customers ledgers); receipts
personally by the judge himself;
warrant already accomplished but still unsigned by for payments received; certificates of stocks and
respondent Judge. securities; contracts, promissory notes and deeds of
3. The determination of the existence of probable
sale; telex and coded messages; business
cause must be made after examination by the judge
At that time respondent Judge was hearing a certain communications, accounting and business records;
of the complainant and the witnesses he may
case; so, by means of a note, he instructed his checks and check stubs; records of bank deposits
produce; and
Deputy Clerk of Court to take the depositions of and withdrawals; and records of foreign remittances,
respondents De Leon and Logronio. After the session covering the years 1966 to 1970."cralaw virtua1aw
4. The warrant must particularly describe the place
had adjourned, respondent Judge was informed that library
to be searched, and the persons or things to be
the depositions had already been taken. The
seized.
stenographer, upon request of respondent Judge, The description does not meet the requirement in Art
read to him her stenographic notes; and thereafter, III, Sec. 1, of the Constitution, and of Sec. 3, Rule 126
respondent Judge asked respondent Logronio to take of the Revised Rules of Court, that the warrant should
the oath and warned him that if his deposition was particularly describe the things to be seized.
It the case at bar, no personal examination at all
found to be false and without legal basis, he could be
was conducted by respondent Judge of the
charged for perjury. Respondent Judge signed
complainant (respondent De Leon) and his witness The search warrant was issued for more than one
respondent de Leons application for search warrant
(respondent Logronio). While it is true that the specific offense.
and respondent Logronios deposition, Search
complainants application for search warrant and the
Warrant No. 2-M-70 was then sign by respondent
witness printed-form deposition were subscribed and
Judge and accordingly issued. The search warrant in question was issued for at
sworn to before respondent Judge, the latter did not
least four distinct offenses under the Tax Code. The
ask either of the two any question the answer to
Three days later, or on February 28, 1970, which was first is the violation of Sec. 46(a), Sec. 72 and Sec. 73
which could possibly be the basis for determining
a Saturday, the BIR agents served the search warrant (the filing of income tax returns), which are
whether or not there was probable cause against
to petitioners at the offices of petitioner corporation interrelated. The second is the violation of Sec. 53
herein petitioners. Indeed, the participants seem to
on Ayala Avenue, Makati, Rizal. Petitioners lawyers (withholding of income taxes at source). The third is
have attached so little significance to the matter that
protested the search on the ground that no formal the violation of Sec. 208 (unlawful pursuit of business
notes of the proceedings before respondent Judge
complaint or transcript of testimony was attached to or occupation); and the fourth is the violation of Sec.
3|Pa g e
Copyright2015-2018 T.P. All rights reserved.
CONSTITUTIONAL LAW II : ARREST SEARCHES AND SEIZURES

209 (failure to make a return of receipts, sales, SECRETARY OF JUSTICE; JOSE LUKBAN, in his intended to be used as the means of committing the
business or gross value of output actually removed or capacity as Acting Director, National Bureau of offense," which is described in the applications
to pay the tax due thereon). Even in their Investigation; SPECIAL PROSECUTORS PEDRO adverted to above as "violation of Central Bank Laws,
classification the six above-mentioned provisions are
embraced in two different titles: Secs. 46(a), 53, 72 D. CENZON, EFREN I. PLANA and MANUEL Tariff and Customs Laws, Internal Revenue (Code)
and 73 are under Title II (Income Tax); while Secs. VILLAREAL, JR. and ASST. FISCAL MANASES G. and the Revised Penal Code."
208 and 209 are under Title V (Privilege Tax on REYES; JUDGE AMADO ROAN, Municipal Court
Business and Occupation). of Manila; JUDGE ROMAN CANSINO, Municipal Petitioner alleged that the aforementioned search
Court of Manila; JUDGE HERMOGENES CALUAG, warrants are null and void, as contravening the
Topic: Who may invoke the Right? Court of First Instance of Rizal-Quezon City Constitution and the Rules of Court because, inter
Branch, and JUDGE DAMIAN JIMENEZ, Municipal alia: (1) they do not describe with particularity the
The Supreme Court made an explanation in this Court of Quezon City, respondents. documents, books and things to be seized; (2) cash
case regarding this right, as qouted:
money, not mentioned in the warrants, were actually
Paredes, Poblador, Cruz and Nazareno and Meer, seized; (3) the warrants were issued to fish evidence
It is next contended by respondents that a Meer and Meer and Juan T. David for petitioners. against the aforementioned petitioners in deportation
corporation is not entitled to protection against
Office of the Solicitor General Arturo A. Alafriz, cases filed against them; (4) the searches and
unreasonable search and seizures. Again, we find no
merit in the contention. Assistant Solicitor General Pacifico P. de Castro, seizures were made in an illegal manner; and (5) the
Assistant Solicitor General Frine C. Zaballero, documents, papers and cash money seized were not
"Although, for the reasons above stated, we are of Solicitor Camilo D. Quiason and Solicitor C. Padua for delivered to the courts that issued the warrants, to
the opinion that an officer of a corporation which is respondents. be disposed of in accordance with law. aid petitioners
charged with a violation of a statute of the state of
filed with the Supreme Court this original action for
its creation, or of an act of Congress passed in the
exercise of its constitutional powers, cannot refuse to CONCEPCION, C.J.: certiorari.
produce the books and papers of such corporation,
we do not wish to be understood as holding
FACTS: Upon application of the officers of the Respondents contended that (1) the contested
that a corporation is not entitled to immunity,
under the 4th Amendment, against government named on the margin hereinafter search warrants are valid and have been issued in
unreasonable searches and seizures. A referred to as Respondents-Prosecutors several accordance with law; (2) that the defects of said
corporation is, after all, but an association of warrants, if any, were cured by petitioners' consent;
individuals under an assumed name and with a judges2 hereinafter referred to as Respondents- and (3) that, in any event, the effects seized are
distinct legal entity. In organizing itself as a Judges issued, on different dates, a total of 42 admissible in evidence against herein petitioners,
collective body it waives no constitutional
immunities appropriate to such body. Its search warrants against petitioners herein 4 and/or regardless of the alleged illegality of the
property cannot be taken without the corporations of which they were officers, directed aforementioned searches and seizures.
compensation. It can only be proceeded to the any peace officer, to search the persons
against by due process of law, and is above-named and/or the premises of their offices, Issue: WON the search warrant is valid
protected, under the 14th Amendment, against
warehouses and/or residences, and to seize and take
unlawful discrimination . . ." (Hale v. Henkel,
201 U.S. 43, 50 L. ed. 652.) possession of the following personal property to wit:
WON a corporation cannot invoke such right

Republic of the Philippines Books of accounts, financial records, vouchers,


Held:1) No. Two points must be stressed in
SUPREME COURT correspondence, receipts, ledgers, journals,
connection with this constitutional mandate, namely:
Manila portfolios, credit journals, typewriters, and other
(1) that no warrant shall issue but upon probable
documents and/or papers showing all business
cause, to be determined by the judge in the manner
G.R. No. L-19550 June 19, 1967 transactions including disbursements receipts,
set forth in said provision; and (2) that the warrant
balance sheets and profit and loss statements and
shall particularly describe the things to be seized.
Bobbins (cigarette wrappers).
HARRY S. STONEHILL, ROBERT P. BROOKS,
JOHN J. BROOKS and KARL BECK, petitioners, None of these requirements has been complied with
vs. as "the subject of the offense; stolen or embezzled
in the contested warrants. Indeed, the same were
HON. JOSE W. DIOKNO, in his capacity as and proceeds or fruits of the offense," or "used or
issued upon applications stating that the natural and
4|Pa g e
Copyright2015-2018 T.P. All rights reserved.
CONSTITUTIONAL LAW II : ARREST SEARCHES AND SEIZURES

juridical person therein named had committed a have no cause of action to assail the legality of the The right of the people to be secure in their persons,
"violation of Central Ban Laws, Tariff and Customs contested warrants and of the seizures made in houses, papers, and effects against unreasonable
Laws, Internal Revenue (Code) and Revised Penal pursuance thereof, for the simple reason that said searches and seizures shall not be violated, and no
Code." In other words, no specific offense had been corporations have their respective personalities, warrants shall issue but upon probable cause, to be
alleged in said applications. The averments thereof separate and distinct from the personality of herein determined by the judge after examination under
with respect to the offense committed were abstract. petitioners, regardless of the amount of shares of oath or affirmation of the complainant and the
As a consequence, it was impossible for the judges stock or of the interest of each of them in said witnesses he may produce, and particularly
who issued the warrants to have found the existence corporations, and whatever the offices they hold describing the place to be searched, and the persons
of probable cause, for the same presupposes the therein may be. Indeed, it is well settled that the or things to be seized.
introduction of competent proof that the party legality of a seizure can be contested only by the
against whom it is sought has performed particular party whose rights have been impaired thereby, and Alvarez vs. The Court of First Instance
acts, or committed specific omissions, violating a that the objection to an unlawful search and seizure
given provision of our criminal laws. As a matter of is purely personal and cannot be availed of by third 64 Phil. 33 (GR No. L-45358)
fact, the applications involved in this case do not parties. Consequently, petitioners herein may not
allege any specific acts performed by herein validly object to the use in evidence against them of
January 29, 1937
petitioners. It would be the legal heresy, of the the documents, papers and things seized from the
highest order, to convict anybody of a "violation of offices and premises of the corporations adverted to
J. Imperial
Central Bank Laws, Tariff and Customs Laws, Internal above, since the right to object to the admission of
Revenue (Code) and Revised Penal Code," as said papers in evidence belongs exclusively to the
Facts:
alleged in the aforementioned applications without corporations, to whom the seized effects belong, and
reference to any determinate provision of said laws may not be invoked by the corporate officers in
or proceedings against them in their individual capacity. On June 3, 1936, the chief of of the secret service of
the Anti-Usury Board presented to Judge David,
presiding judge of CFI of Tayabas, alleging that
To uphold the validity of the warrants in question With respect to the documents, papers and things according to reliable information, the petitioner is
would be to wipe out completely one of the most seized in the residences of petitioners herein, the keeping in his house in Infanta, Tayabas documents,
receipts, lists, chits and other papers used by him in
fundamental rights guaranteed in our Constitution, aforementioned resolution of June 29, 1962, lifted the
connection with his activities as a money lender
for it would place the sanctity of the domicile and the writ of preliminary injunction previously issued by charging usurious rates of interest in violation of the
privacy of communication and correspondence at the this Court, thereby, in effect, restraining herein law.
mercy of the whims caprice or passion of peace Respondents-Prosecutors from using them in
officers. This is precisely the evil sought to be evidence against petitioners herein. In his oath the chief of the secret service did not
remedied by the constitutional provision above swear to the truth of his statements upon his
quoted to outlaw the so-called general warrants. It knowledge of the facts but the information received
In connection with said documents, papers and
is not difficult to imagine what would happen, in by him from a reliable person. Upon this questioned
things, two (2) important questions need be settled, affidavit, the judge issued the search warrant,
times of keen political strife, when the party in power namely: (1) whether the search warrants in question, ordering the search of the petitioners house at any
feels that the minority is likely to wrest it, even and the searches and seizures made under the time of the day or night, the seizure of the books and
though by legal means. authority thereof, are valid or not, and (2) if the documents and the immediate delivery of such to
him (judge). With said warrant, several agents of the
answer to the preceding question is in the negative,
Anti-Usury Board entered the petitioner's store and
Thus, the documents, papers, and things seized whether said documents, papers and things may be residence at 7 o'clock of the night and seized and
under the alleged authority of the warrants in used in evidence against petitioners herein. took possession of various articles belonging to the
question may be split into two (2) major groups, petitioner.
namely: (a) those found and seized in the offices of Petitioners maintain that the aforementioned search
the aforementioned corporations, and (b) those warrants are in the nature of general warrants and The petitioner asks that the warrant of issued by the
found and seized in the residences of petitioners Court of First Instance of Tayabas, ordering the
that accordingly, the seizures effected upon the
herein. search of his house and the seizure, at anytime of
authority there of are null and void. In this the day or night, of certain accounting books,
connection, the Constitution provides: documents, and papers belonging to him in his
2) The Supreme Court held that petitioners herein residence situated in Infanta, Tayabas, as well as the
5|Pa g e
Copyright2015-2018 T.P. All rights reserved.
CONSTITUTIONAL LAW II : ARREST SEARCHES AND SEIZURES

order of a later date, authorizing the agents of the absence or probable cause, the manner in which the who executed the warrant was thereby placed in a
Anti-Usury board to retain the articles seized, be search and seizure was made, the place or thing position enabling him to identify the articles, which
declared illegal and set aside, and prays that all the searched, and the character of the articles procured. he did.
articles in question be returned to him.
Neither the Constitution nor General Orders. No. 58 At the hearing of the incidents of the case raised
Issues: provides that it is of imperative necessity to take the before the court it clearly appeared that the books
deposition of the witnesses to be presented by the and documents had really been seized to enable the
applicant or complainant in addition to the affidavit Anti-Usury Board to conduct an investigation and
Whether or not there is a valid search warrant
of the latter. The purpose of both in requiring the later use all or some of the articles in question as
presentation of depositions is nothing more than to evidence against the petitioner in the criminal cases
Held: satisfy the committing magistrate of the existence of that may be filed against him. The seizure of books
probable cause. Therefore, if the affidavit of the and documents by means of a search warrant, for the
applicant or complainant is sufficient, the judge may purpose of using them as evidence in a criminal case
A search warrant is an order in writing, issued in the
dispense with that of other witnesses. Inasmuch as against the person in whose possession they were
name of the People of the Philippine Islands, signed
the affidavit of the agent in this case was insufficient found, is unconstitutional because it makes the
by a judge or a justice of the peace, and directed to a
because his knowledge of the facts was not personal warrant unreasonable, and it is equivalent to a
peace officer, commanding him to search for
but merely hearsay, it is the duty of the judge to violation of the constitutional provision prohibiting
personal property and bring it before the court. Of all
require the affidavit of one or more witnesses for the the compulsion of an accused to testify against
the rights of a citizen, few are of greater importance
purpose of determining the existence of probable himself. Therefore, it appearing that at least nineteen
or more essential to his peace and happiness than
cause to warrant the issuance of the search warrant. of the documents in question were seized for the
the right of personal security, and that involves the
When the affidavit of the applicant of the complaint purpose of using them as evidence against the
exemption of his private affairs, books, and papers
contains sufficient facts within his personal and direct petitioner in the criminal proceeding or proceedings
from the inspection and scrutiny of others. While the
knowledge, it is sufficient if the judge is satisfied that for violation against him, we hold that the search
power to search and seize is necessary to the public
there exist probable cause; when the applicant's warrant issued is illegal and that the documents
welfare, still it must be exercised and the law
knowledge of the facts is mere hearsay, the affidavit should be returned to him.
enforced without transgressing the constitutional
of one or more witnesses having a personal
rights or citizen, for the enforcement of no statue is
knowledge of the fact is necessary. We conclude,
of sufficient importance to justify indifference to The Anti-Usury Board insinuates in its answer that
therefore, that the warrant issued is likewise illegal
thebasis principles of government. the petitioner cannot now question the validity of the
because it was based only on the affidavit of the
search warrant or the proceedings had subsequent to
agent who had no personal knowledge of the facts.
the issuance thereof, because he has waived his
As the protection of the citizen and the maintenance
constitutional rights in proposing a compromise
of his constitutional right is one of the highest duties
Section 101 of General Orders, No. 58 authorizes that whereby he agreed to pay a fine of P200 for the
and privileges of the court, these constitutional
the search be made at night when it is positively purpose of evading the criminal proceeding or
guaranties should be given a liberal construction or a
asserted in the affidavits that the property is on the proceedings. We are of the opinion that there was no
strict construction in favor of the individual, to
person or in the place ordered to be searched. As we such waiver, first, because the petitioner has
prevent stealthy encroachment upon, or gradual
have declared the affidavits insufficient and the emphatically denied the offer of compromise and,
depreciation on, the rights secured by them. Since
warrant issued exclusively upon it illegal, our second, because if there was a compromise it
the proceeding is a drastic one, it is the general rule
conclusion is that the contention is equally well reffered but to the institution of criminal proceedings
that statutes authorizing searches and seizure or
founded and that the search could not legally be fro violation of the Anti-Usury Law. The waiver would
search warrants must be strictly construed.
made at night. have been a good defense for the respondents had
the petitioner voluntarily consented to the search
Unreasonable searches and seizures are a menace and seizure of the articles in question, but such was
The only description of the articles given in the
against which the constitutional guarantee afford full not the case because the petitioner protested from
affidavit presented to the judge was as follows: "that
protection. The term "unreasonable search and the beginning and stated his protest in writing in the
there are being kept in said premises books,
seizure" is not defined in the Constitution or in insufficient inventory furnished him by the agents.
documents, receipts, lists, chits and other papers
General Orders No. 58, and it is said to have no fixed,
used by him in connection with his activities as
absolute or unchangeable meaning, although the
money-lender, charging a usurious rate of interest, in Jose Burgos vs. Chief of Staf
term has been defined in general language. All illegal
violation of the law." Taking into consideration the
searches and seizure are unreasonable while lawful
nature of the article so described, it is clear that no
ones are reasonable. What constitutes a reasonable G.R. No L-64261
other more adequate and detailed description could
or unreasonable search or seizure in any particular
have been given, particularly because it is difficult to
case is purely a judicial question, determinable from
give a particular description of the contents thereof. December 26, 1984
a consideration of the circumstances involved,
The description so made substantially complies with
including the purpose of the search, the presence or
the legal provisions because the officer of the law
6|Pa g e
Copyright2015-2018 T.P. All rights reserved.
CONSTITUTIONAL LAW II : ARREST SEARCHES AND SEIZURES

Facts: Probable cause for a search is defined as such facts of Act No. 2333, entitled "An Act Relative to
and circumstances which would lead a reasonably Untrue, Deceptive and Misleading
discreet and prudent man to believe that an offense Advertisements," as amended by Act No.
Two warrants were issued against petitioners for the
has been committed and that the objects sought in 3740.
search on the premises of Metropolitan Mail and
connection with the offense are in the place sought
We Forum newspapers and the seizure of items
to be searched.
alleged to have been used in subversive activities. After appropriate proceedings, the
Petitioners prayed that a writ of preliminary investigating prosecutor released a Joint
mandatory and prohibitory injunction be issued for The Court ruled that the affidavits submitted for the Resolution where he recommended the filing
the return of the seized articles, and that application of the warrant did not satisfy the of an information against the petitioners and
respondents be enjoined from using the articles thus requirement of probable cause, the statements of the others for the violation of Article 318 of the
seized as evidence against petitioner. witnesses having been mere generalizations. Revised Penal Code (OTHER DECEITS) and
the dismissal of the complaints for the
Petitioners questioned the warrants for the lack of Furthermore, jurisprudence tells of the prohibition on violation of Article 315, 2(d) of the Revised
probable cause and that the two warrants issued the issuance of general warrants. (Stanford vs. State Penal Code; R.A. No. 7394; Act No. 2333, as
indicated only one and the same address. In addition, of Texas). The description and enumeration in the amended by Act No. 3740; and E.O. No. 913.
the items seized subject to the warrant were real warrant of the items to be searched and seized did
properties. not indicate with specification the subversive nature City Prosecutor Candido V. Rivera approved
of the said items. the recommendation with the modification
Issue: that Rosemarie Vera, Quintin Gomez, Jr., and
G.R. No. 113930 March 5, 1996 Chito Gonzales be excluded from the charge
on the ground of insufficiency of evidence.
Whether or not the two warrants were valid to justify
seizure of the items. PAUL G. ROBERTS, JR., RODOLFO C. SALAZAR,
LUIS LORENZO, SR., LUIS LORENZO, JR., the petitioners filed with the Office of the
AMAURY R. GUTIERREZ, BAYANI N. FABIC, JOSE City Prosecutor a motion for the
Held: reconsideration of the Joint
YULO, JR., ESTEBAN B. PALANNUAYAN, and
WONG FONG FUI, petitioners, Resolution alleging therein that (a) there
The defect in the indication of the same address in vs. was neither fraud in the Number Fever
the two warrants was held by the court as a THE COURT OF APPEALS, THE HON. MAXIMIANO Promotion nor deviation from or
typographical error and immaterial in view of the ASUNCION, in his capacity as the Presiding modification of the promotional rules
correct determination of the place sought to be Judge of the Regional Trial Court, Quezon City, approved by the Department of Trade and
searched set forth in the application. The purpose Branch 104, HON. APOLINARIO G. EXEVEA, Industry (DTI), for from the start of the
and intent to search two distinct premises was HON. HENRICK F. GINGOYON, and HON. PHILIP promotion, it had always been clearly
evident in the issuance of the two warrant. A. AGUINALDO, in their capacities as Members explained to the public that for one to be
of the Department of Justice "349" Committee, entitled to the cash prize his crown must
and the CITY PROSECUTOR OF QUEZON CITY, bear both the winning number and the
As to the issue that the items seized were real correct security code as they appear in the
respondents.
properties, the court applied the principle in the case DTI list; (b) the complainants failed to
of Davao Sawmill Co. v. Castillo, ruling that allege, much less prove with prima
FACTS: facie evidence, the specific overt criminal
machinery which is movable by nature becomes
immobilized when placed by the owner of the acts or omissions purportedly committed by
tenement, property or plant, but not so when placed Several thousand holders 6 of "349" Pepsi each of the petitioners; (c) the compromise
by a tenant, usufructuary, or any other person crowns in connection with the Pepsi Cola agreement entered into by PEPSI is not an
having only a temporary right, unless such person Products Phils., Inc.'s (PEPSI's) Number admission of guilt; and (d) the evidence
acted as the agent of the owner. In the case at bar, Fever Promotion 7 filed with the Office of the establishes that the promo was carried out
petitioners did not claim to be the owners of the land City Prosecutor of Quezon City complaints with utmost good faith and without
and/or building on which the machineries were against the petitioner's malicious intent.
placed. This being the case, the machineries in
question, while in fact bolted to the ground remain
The complaints respectively accuse the petitioners filed with the DOJ a Petition for
movable property susceptible to seizure under a
petitioners and the other PEPSI officials of Review
search warrant.
the following crimes: (a) estafa; (b) violation
of R.A. No. 7394, otherwise known as the Petitioners filed Motions to Suspend
However, the Court declared the two warrants null Consumer Act of the Philippines; (c)
and void. Proceedings and to Hold in Abeyance
violation of E.O. No. 913; 8 and (d) violation
7|Pa g e
Copyright2015-2018 T.P. All rights reserved.
CONSTITUTIONAL LAW II : ARREST SEARCHES AND SEIZURES

Issuance of Warrants of Arrest on the ground express declaration" in the questioned order cases covered by the rule on summary
that they had filed the aforesaid Petition for as to the basis of such finding does not give procedure where the accused fails to appear
Review. rise to an adverse inference, for the when required; and in cases filed with them
respondent Judge enjoys in his favor the which are cognizable by the Regional Trial
presumption of regularity in the Courts (RTCs);and
Private prosecutor Julio Contreras filed
performance of his official duty.
an Ex-Parte Motion for Issuance of Warrants
of Arrest. (2) by the Metropolitan Trial Courts in the
The Court of Appeals then issued a National Capital Region (MeTCs-NCR) and
resolution denying the application for a writ the RTCs in cases filed with them after
In the afternoon of that same day, petitioner appropriate preliminary investigations
of preliminary injunction.
Paul Roberts, Jr., filed a Supplemental Urgent conducted by officers authorized to do so
Motion to Hold in Abeyance Issuance of other than judges of MeTCs, MTCs and
Warrant of Arrest and to Suspend HELD: MCTCs.
Proceedings. He stressed that the DOJ had
taken cognizance of the Petition for Review ON CRESPO v. MOGUL
by directing the City Prosecutor to elevate As to the first, a warrant can issue only if
the records the judge is satisfied after an examination
There is nothing in Crespo vs. Mogul which in writing and under oath of the
bars the DOJ from taking cognizance of an complainant and the witnesses, in the
The next day, respondent Judge Asuncion form of searching questions and
appeal, by way 'of a petition for review, by
issued an order advising the parties that his an accused in a criminal case from an answers, that a probable cause exists and
court would "be guided by the doctrine laid unfavorable ruling of the investigating that there is a necessity of placing the
down by the Supreme Court in the case prosecutor. It merely advised the DOJ to, "as respondent under immediate custody in
of Crespo vs. Mogul, and not by the far as practicable, refrain from entertaining order not to frustrate the ends of justice.
resolution of the Department of Justice on a petition for review or appeal from the
the petition for review undertaken by the action of the fiscal, when the complaint or
accused." As to the second, this Court held in Soliven
information has already been filed in Court."
vs. Makasiar 63 that the judge is not
required to personally examine the
respondent Judge Asuncion issued the Crespo could not have intended otherwise complainant and the witnesses, but
challenged order (1) denying the petitioners' without doing violence to, or repealing, the
Motion to Suspend Proceedings and to Hold last paragraph of Section 4, Rule 112 of the
in Abeyance Issuance of Warrants of Arrest [f]ollowing established doctrine and
Rules of Court 54 which recognizes the procedure, he shall:
and the public prosecutor's Motion to Defer authority of the Secretary of Justice to
Arraignment and (2) directing the issuance reverse the resolution of the provincial or
of the warrants of arrest city prosecutor or chief state prosecutor (1) personally evaluate the report
upon petition by a proper party. and supporting documents
Upon receipt of the original records of the submitted by the fiscal regarding
criminal case, the Court of Appeals found the existence of probable cause
Section 2, Article III of the present and, on the basis thereof; issue a
that a copy of the Joint Resolution had in Constitution provides that no search warrant
fact been forwarded to, and received by, the warrant of arrest; or
or warrant of arrest shall issue except upon
trial court on 22 April 1993, which fact probable cause to be determined personally
belied the petitioners' claim that the by the judge after examination under oath (2) if on the basis thereof he finds
respondent Judge had not the slightest basis or affirmation of the complainant and the no probable cause, he may
at all for determining probable cause when witnesses he may produce. disregard the fiscal's report and
he ordered the issuance of warrants of require the submission of
arrest. supporting affidavits of witnesses
Under existing laws, warrants of arrest may to aid him in arriving at a
be issued: conclusion as to the existence of
It ruled that the Joint Resolution "was
probable cause.
sufficient in itself to have been relied upon
by respondent Judge in convincing himself (1) by the Metropolitan Trial Courts (MeTCs)
that probable cause indeed exists for the except those in the National Capital Region, PROBABLE CAUSE
purpose of issuing the corresponding Municipal Trial Courts (MTCs), and Municipal
warrants of arrest"; and that the "mere Circuit Trial Courts (MCTCs) in cases falling
silence of the records or the absence of any within their exclusive original jurisdiction; in In satisfying himself of the existence of a
8|Pa g e
Copyright2015-2018 T.P. All rights reserved.
CONSTITUTIONAL LAW II : ARREST SEARCHES AND SEIZURES

probable cause for the issuance of a warrant certification which are material in issuance of a warrant of arrest. The
of arrest, the judge, following established assisting the Judge to first kind of preliminary
doctrine and procedure, may either: make hisdetermination. investigation is executive in nature.
It is part of the prosecution's job.
The second kind of preliminary
(a) Rely upon the fiscal's The teachings then
investigation which is more
certification of the existence of of Soliven, Inting, Lim, Allado, properly called preliminary
probable cause whether or not and Webb reject the proposition that the examination is judicial in nature
the case is cognizable only by investigating prosecutor's certification in an and is lodged with the judge.
the Regional Trial Court and on information or his resolution which is made
the basis thereof, issue a warrant the basis for the filing of the information, or
of arrest. . . . both, would suffice in the judicial People vs. Chua Ho San [GR 128222, 17 June
determination of probable cause for the 1999] En Banc, Davide Jr. (CJ): 13 concur, 1 on
issuance of a warrant of arrest. leave
This requirement of evaluation not only of
the report or certification of the fiscal but
also of the supporting documents was Unfortunately, nothing accompanied the Facts: In response to reports of rampant smuggling
further explained in People vs. Inting, where information upon its filing on 12 April 1993 of firearms and other contraband, Jim Lagasca Cid, as
this Court specified what the documents with the trial court. As found by the Court of Chief of Police of the Bacnotan Police Station, of La
may consist of, viz., "the affidavits, the Appeals in its resolution of 1 July 1993, a Union began patrolling the Bacnotan coastline with
transcripts of stenographic notes (if copy of the Joint Resolution was his officers. While monitoring the coastal area of
any), and all other supporting forwarded to, and received by, the trial Barangay Bulala on 29 March 1995, he intercepted a
documents behind the Prosecutor's court only on 22 April 1993. radio call at around 12:45 p.m. from Barangay
certification which are material in Captain Juan Almoite of Barangay Tammocalao
assisting the Judge to make his requesting police assistance regarding an unfamiliar
determination" of probable cause. Thus: no affidavits of the witnesses, transcripts of speedboat the latter had spotted, which looked
stenographic notes of the proceedings different from the boats ordinarily used by fisherfolk
during the preliminary investigation, or of the area and was poised to dock at Tammocalao
We emphasize the important other documents submitted in the course shores. Cid and 6 of his men led by his Chief
features of the constitutional thereof were found in the records Investigator, SPO1 Reynoso Badua, proceeded
mandate that ". . . no search forthwith to Tammocalao beach, conferred with
warrant or warrant of arrest shall Almoite, and observed that the speedboat ferried a
issue except upon probable cause Clearly, when respondent Judge Asuncion
lone male passenger. When the speedboat landed,
to be determined personally by the issued the assailed order of 17 May 1993
the male passenger alighted, and using both hands,
judge . . ." (Article III, Section 2, directing, among other things, the issuance
carried what appeared a multicolored strawbag, and
Constitution). of warrants of arrest, he had only the
walked towards the road. By this time, Almoite, Cid
information, amended information, and Joint
and Badua, the latter two conspicuous in their
Resolution as bases thereof. He did not have
First, the determination of probable uniform and issued side-arms, became suspicious of
the records or evidence supporting the
cause is a function of the Judge. It the man as he suddenly changed direction and broke
prosecutor's finding of probable cause. .
is not for the Provincial Fiscal or into a run upon seeing the approaching officers.
Prosecutor nor the Election Badua, prevented the man from fleeing by holding on
Supervisor to ascertain. Only the NOTES: to his right arm. Although Cid introduced themselves
Judge and the Judge alone makes as police officers, the man appeared impassive.
this determination. Speaking in English, then in Tagalog, and later in
In criminal prosecutions, the determination
Ilocano, Cid then requested the man to open his bag,
of probable cause may either be an but he seemed not to understand. Cid then resorted
Second, the preliminary inquiry executive or a judicial prerogative. to "sign language," motioning with his hands for the
made by a Prosecutor does not man to open the bag. The man apparently
bind the Judge. It merely assists understood and acceded to the request. A search of
We reiterate that preliminary
him to make the determination of the bag yielded several transparent plastic packets
investigation should be
probable cause. By itself, the containing yellowish crystalline substances. As Cid
distinguished as to whether it is an
Prosecutor's certification of wished to proceed to the police station, he signaled
investigation for the determination
probable cause is ineffectual. It is the man to follow, but the latter did not comprehend.
of a sufficient ground for the filing
the report, the affidavits, the Hence, Cid placed his arm around the shoulders of
of the information or it is an
transcripts of stenographic notes (if the man and escorted the latter to the police
investigation for the determination
any), and all other supporting headquarters. At the police station, Cid then "recited
of a probable cause for the
documents behind the Prosecutor's
9|Pa g e
Copyright2015-2018 T.P. All rights reserved.
CONSTITUTIONAL LAW II : ARREST SEARCHES AND SEIZURES

and informed the man of his constitutional rights" to methamphetamine hydrochloride without legal suspicious behavior, i.e. he attempted to flee when
remain silent, to have the assistance of a counsel, authority to do so. Chua prays for the reversal of the he saw the police authorities, and the apparent ease
etc. Eliciting no response from the man, Cid ordered RTC decision and his acquittal before the Supreme by which Chua can return to and navigate his
his men to find a resident of the area who spoke Court. speedboat with immediate dispatch towards the high
Chinese to act as an interpreter. In the meantime, seas, do not constitute "probable cause." None of the
Badua opened the bag and counted 29 plastic telltale clues, e.g., bag or package emanating the
Issue: Whether persistent reports of rampant
packets containing yellowish crystalline substances. pungent odor of marijuana or other prohibited drug,
smuggling of firearm and other contraband articles,
The interpreter, Mr. Go Ping Guan, finally arrived, 20 confidential report and/or positive identification
Chua's watercraft differing in appearance from the
through whom the man was "apprised of his by informers of courier(s) of prohibited drug and/or
usual fishing boats that commonly cruise over the
constitutional rights." When the policemen asked the the time and place where they will transport/deliver
Bacnotan seas, Chuas illegal entry into the
man several questions, he retreated to his obstinate the same, suspicious demeanor or behavior and
Philippines, Chuas suspicious behavior, i.e. he
reticence and merely showed his ID with the name suspicious bulge in the waist accepted by the
attempted to flee when he saw the police authorities,
Chua Ho San printed thereon. Chua's bag and its Court as sufficient to justify a warrantless arrest
and the apparent ease by which Chua can return to
contents were sent to the PNP Crime Laboratory at exists in the case. There was no classified
and navigate his speedboat with immediate dispatch
Camp Diego Silang, Carlatan, San Fernando, La Union information that a foreigner would disembark at
towards the high seas, constitute "probable cause."
for laboratory examination. In the meantime, Chua Tammocalao beach bearing prohibited drug on the
was detained at the Bacnotan Police Station. Later, date in question. Chua was not identified as a drug
Police Chief Inspector and Forensic Chemist Theresa Held: No. Enshrined in the Constitution is the courier by a police informer or agent. The fact that
Ann Bugayong Cid (wife of Cid), conducted a inviolable right to privacy of home and person. It the vessel that ferried him to shore bore no
laboratory examination of 29 plastic packets, adn in explicitly ordains that people have the right to be resemblance to the fishing boats of the area did not
her Chemistry Report D-025-95, she stated that her secure in their persons, houses, papers and effects automatically mark him as in the process of
qualitative examination established the contents of against unreasonable searches and seizures of perpetrating an offense. The search cannot therefore
the plastic packets, weighing 28.7 kilos, to be whatever nature and for any purpose. Inseparable, be denominated as incidental to an arrest. To
positive of methamphetamine hydrochloride or and not merely corollary or incidental to said right reiterate, the search was not incidental to an arrest.
shabu, a regulated drug. Chua was initially charged and equally hallowed in and by the Constitution, is There was no warrant of arrest and the warrantless
with illegal possession of methamphetamine the exclusionary principle which decrees that any arrest did not fall under the exemptions allowed by
hydrochloride before the RTC (Criminal Case 4037). evidence obtained in violation of said right is the Rules of Court as already shown. From all
However, pursuant to the recommendation of the inadmissible for any purpose in any proceeding. The indications, the search was nothing but a fishing
Office of the Provincial Prosecutor of San Fernando, Constitutional proscription against unreasonable expedition. Casting aside the regulated substance as
La Union, the information was subsequently searches and seizures does not, of course, forestall evidence, the same being the fruit of a poisonous
amended to allege that Chua was in violation of reasonable searches and seizure. This interdiction tree, the remaining evidence on record are
Section 15, Article III of RA 6425 as amended by RA against warrantless searches and seizures, however, insufficient, feeble and ineffectual to sustain Chuas
7659 (illegal transport of a regulated drug). At his is not absolute and such warrantless searches and conviction.
arraignment on 31 July 1995, where the amended seizures have long been deemed permissible by
complaint was read to him by a Fukien-speaking jurisprudence. The Rules of Court recognize
People vs Molina
interpreter, Chua entered a plea of not guilty. Trial permissible warrantless arrests, to wit: (1) arrests in
finally ensued, with interpreters assigned to Chua flagrante delicto, (2) arrests effected in hot pursuit,
(upon the RTC's direct request to the Taipei Economic and (3) arrests of escaped prisoners. The prosecution 19 February 2001 | Ponente: Ynares-Santiago
and Cultural Office in the Philippines, after its failure and the defense painted extremely divergent
to acquire one from the Department of Foreign versions of the incident, but the Court is certain that
Affairs). Chua provided a completely different story, Chua was arrested and his bag searched without the Overview: SPO1 Paguidopon received a tip about
claiming that the bags belong to his employer Cho benefit of a warrant. There are no facts on record drug pushers. He previously caught a glimpse of one
Chu Rong, who he accompanied in the speedboat; reasonably suggestive or demonstrative of Chuas of them, Mula, so he was able to point to him and his
that they decided to dock when they were low on fuel participation in an ongoing criminal enterprise that companion, Molina, to arresting officers when they
and telephone battery; that the police, with nary any could have spurred police officers from conducting were aboard a trisikad. Upon accosting them, the
spoken word but only gestures and hand movements, the obtrusive search. The RTC never took the pains of police were able to find marijuana in a bag carried by
escorted him to the precinct where he was pointing to such facts, but predicated mainly its Molina, leading to their arrest. The court however
handcuffed and tied to a chair; that the police, led by decision on the finding that "accused was caught held that they were illegally arrested because their
an officer, arrived with the motor engine of the red-handed carrying the bagful of shabu when case dont fall under the exception of an in flagrante
speedboat and a bag, which they presented to him; apprehended." In short, there is no probable cause. delicto arrest, there being no outward indication that
that the police inspected opened the bag, weighed Persistent reports of rampant smuggling of firearm could justify their arrest.
the contents, then proclaimed them as and other contraband articles, Chua's watercraft
methamphetamine hydrochloride. In a decision differing in appearance from the usual fishing boats Statement of the Case:
promulgated on 10 February 1997, the RTC convicted that commonly cruise over the Bacnotan seas,
Chua for transporting 28.7 kilos of Chuas illegal entry into the Philippines, Chuas
- This is for review of the decision of the RTC
10 | P a g e
Copyright2015-2018 T.P. All rights reserved.
CONSTITUTIONAL LAW II : ARREST SEARCHES AND SEIZURES

findingNasario Molina alias "Bobong" and inadmissible. The trial court denied this. The committed, is actually committing, or is
Gregorio Mula alias "Boboy" guilty of two waived presentation of evidence, and attempting to commit an offense
violation of Sec. 8 of RA 6245, or the opted to file a joint memorandum. Later, the
Dangerous Drugs Act, by possessing 946.9 trial court still found them guilty, and
(b) Arrest efected in hot pursuit - when
grants of dried marijuana. sentenced them to suffer the death penalty.
an offense has just been committed and
he has probable cause to believe based
- Molina and Mula pleaded guilty upon - Pursuant to Art. 47 of the RPC and Rule 122, on personal knowledge of facts or
arraingnment. Sec. 10 of the ROC, the case is elevated to circumstances that the person to be
the SC on automatic review. arrested has committed it.
Statement of Facts:
- The SolGen moved for the acquittal of the (c) Arrest of escaped prisoners - when
two. the person to be arrested is a prisoner
- On June 1996, SPO1 Marino Paguidopon
who has escaped from penal
received information about a marijuana
establishment or a place where he is
pusher in Davao. Paguidopon first saw the Issue and Held:
serving final judgment or is temporarily
pusher in person on July of the same year,
confined while his case is pending, or
when his informer identified Mula as the
- Was the arrest of Mula and Molina fall under has escaped while being transferred
driver of a motorcylce who just passed by
the exception of in flagrante delicto in from one confinement to another.
them. Molina, on the other hand, was never
warrantless arrests? NO
identified prior arrest.
- In this case, the trial court found that the
Applicable Laws: warrantless arrest and seizure were valid
- In the morning of August 8, 1996,
apparently because they were caught in
Paguidopon received information that the
flagrante delicto in possession of the prohibited
drug pushers will pass by at NHA, Ma-a, Article III, Sec. 2
drugs. But the question is: does the
Davao City that morning, so he called for
present case aptly fall within the
assistance from the PNP. A team composed
Article III, Sec. 3 exceptions to the warrant requirement? In
of SPO4 Cloribel, SPO2 Paguidopon (brother
in flagrante delicto arrests, it is settled that
of Marino), and SPO1 Pamplona were
"reliable information" alone is not
dispatched to proceed to Marino's housee Rationale:
sufficient to constitute probable cause that
where they'll wait for the drug pushers will
would justify in flagrante delicto arrests.
pass by.
- The law mandates that searches be carried
out with a search warrant upon the
o People vs Chua Ho San: The arresting
- Two hours later, a "trisikad" identified by existence of probable cause. Likewise, the
officer must have personal knowledge
Paguidopon as carrying Molina and Mula law protects against unreasonable searches
that the person he is arresting has
passed by. So, the team boarded their and seizures and holds evidence taken from
committed, is committing or is about to
vehicle, overtook the trisikad and accosted such incidents as inadmissible as evidence.
commit the offense.
the two.
- There are exceptions to this, the first being
o People vsAminnudin: The accused was
- At that point, Mula was holding a black bag. seizure conducted incidental to a lawful
He handed the same to Molina. Pamplona, arrest. just disembarking the vessel. He only
introducing himself as a police officer, asked became suspect when the informer
Molina to open the bag, to which Molina pointed him to the officials.
- For this, there should be a lawful arrest first,
replied "Boss, if possible, we will settle this."
before a search can be made. It doesn't
work the other way around. o People vsMengote: eyes darting from
- Pamplona insisted on opening the bag, side to side while holding one's
which revealed dried marijuana leaves abdomen, is not indicative of probable
- Likewise, as a rule, an arrest is legitimate if cause to suspect the accused of
inside. Thereafter, Mula and Molina were
it's with a valid warrant of arrest. However, committing the offense.
handcuffed.
a police officer may conduct warrantless
arrests:
- Mula and Molina filed a Demurrer to o People vsEncinada: riding a motorela
Evidence, saying that the marijuna was while holding two plastic baby chairs is
(a) In flagrante delicto - when, in his also not indicative of probable cause
illegally seized from them, therefore it is
presence, the person to be arrested has
11 | P a g e
Copyright2015-2018 T.P. All rights reserved.
CONSTITUTIONAL LAW II : ARREST SEARCHES AND SEIZURES

o Malacatvs CA: "standing on the corner right against unreasonable searches and seizure paraphernalia was in fact seized by the police. The
with his eyes moving very fast and just because they had an implied acquiescence fact is that none was taken by virtue of the search
looking at every person that come to the search. This was mere passive conformity warrant issued. If at all, therefore, the search warrant
nearer to them" is not an act given the circumstance. is void only insofar as it authorized the seizure of
evidencing that the accused is drug paraphernalia, but it is valid as to the seizure of
attempting to commit a crime. There methamphetamine hydrochloride as to which
Judgment: Mula and Molina are acquitted. Both the
was no ground to even believe that the evidence was presented showing probable cause as
arrest and the seizure are illegal.
accused was armed with a weapon to its existence. In sum, with respect to the seizure of
since even a telltale bulge at the front shabu from Salanguits residence, Search Warrant
waistline of the accused wouldn't be People vs ROBERTO SALANGUIT y KO 160 was properly issued, such warrant being founded
visible from the officers' view. on probable cause personally determined by the
judge under oath or affirmation of the deposing
FACTS: witness and particularly describing the place to be
- To constitute in flagrante delcito arrests,
searched and the things to be seized. With respect
there are two requisites:
A search warrant was shown to the accused- to, and in light of the plain view doctrine, the police
appellant and the police operatives started searching failed to allege the time when the marijuana was
1. The person to be arrested must execute the house. They found heat-sealed transparent found, i.e., whether prior to, or contemporaneous
an overt act indicating that he has just plastic bags containing a white crystalline substance, with, the shabu subject of the warrant, or whether it
committed, is actually committing, or is a paper clip box also containing a white crystalline was recovered on Salanguits person or in an area
attempting to commit a crime substance, and two bricks of dried leaves which within his immediate control. Its recovery, therefore,
appeared to be marijuana. A receipt of the items presumably during the search conducted after the
seized was prepared, but the accused-appellant shabu had been recovered from the cabinet, as
2. Such overt act is done in the presence
refused to sign it. Charges against Roberto Salanguit attested to by SPO1 Badua in his deposition, was
or within the view of the arresting
y Ko for violations of Republic Act (RA) 6425, i.e. for invalid. Thus, the Court affirmed the decision as to
officer.
possession of shabu and marijuana, (Criminal Cases Criminal Case Q-95-64357 only.
Q-95-64357 and Q-95-64358, respectively) were
filed, and after hearing, the trial court convicted him Microsoft corp. v. Maxicorp
in Criminal Cases Q-95-64357 and Q-95-64358 for
- In the present case, Mula and Molina violation of Section 16 and 8, respectively.
G.R No. 140946
manifested no outward indication to justify
their arrest. Holding a bag, Molina saying The accused-appellant contended that the evidence
"Boss, if possible we will settle this", which against him was inadmissible because the warrant 438 SCRA 224 Mercantile Law Intellectual
allegedly arouse the suspicion of the officers, are used in obtaining it was invalid. Property Law on Copyright Probable Cause
not constitutive of probable cause. Were it not in Issuing Search Warrant
for Paguidopon, the accused won't even be
identified nor found suspicious. ISSUES:
FACTS:In 1996, DominadorSamiano, Jr., an agent of
the National Bureau of Investigation (NBI) conducted
- As to Mula and Molina's identity, Paguidopon Whether the warrant was invalid for failure of a surveillance against Maxicorp, Inc. He observed
said that he conducted a surveillance of Mula, providing evidence to support the seizure of drug that MicrosoftSoftwares (Windows Operating
and caught a glimpse of him on the road. paraphernalia, and whether the marijuana may be Systems) were being produced and packaged within
However, he doesn't even know his name or included as evidence in light of the plain view the premises of Maxicorp. Samiano, together with a
address. As to Molina, he admits not seeing him doctrine. civilian witness (John Benedict Sacriz) then bought a
before the arrest. With this, Pamplona's claim computer unit from Maxicorp. The unit was pre-
that he knew of the name of the accused is HELD: installed with a pirated copy of Windows. For their
baseless. purchase, they were issued a receipt, however, the
receipt was in the name of a certain Joel Diaz.
Yes. The warrant authorized the seizure of Subsequently, Samiano applied for a search warrant
- This case is different from People vsEncinada, undetermined quantity of shabu and drug before the RTC. He brought with him Sacriz as
because there the officers knew the identity of paraphernalia. Evidence was presented showing witness. He also brought the computer unit they
the person to be arrested. Nevertheless the probable cause of the existence of bought as evidence as well as the receipt. He even
arrest was still held illegal since the accused did methamphetamine hydrochloride or shabu. The fact added an additional witness (FelixbertoPante), a
not show any suspicious behavior. that there was no probable cause to support the computer technician, who showed the judge that
application for the seizure of drug paraphernalia does the software in the computer unit bought by Samiano
- It can't be said that the accused waived their not warrant the conclusion that the search warrant is from Maxicorp was pirated. The RTC judge,
void. This fact would be material only if drug
12 | P a g e
Copyright2015-2018 T.P. All rights reserved.
CONSTITUTIONAL LAW II : ARREST SEARCHES AND SEIZURES

convinced that there is a probable cause for a case of determine probable cause. On November 21, 1989, petitioners Vicente Lim, Sr.
copyright infringement and unfair competition and Susana Lim filed with us a verified petition for
committed by Maxicorp, issued the corresponding Held: No change of venue w/c was authorized, from the RTC of
warrant. Maxicorp assailed the legality of the warrant Ratio: In satisfying himself of the existence of Masbate to the RTCt of Makati to avoid miscarriage of
before the Court of Appeals. The Court of Appeals probable cause to issue a warrant of arrest, the judge justice. The cases were raffled to Branch 56 presided
ruled in favor of Maxicorpand in its decision it isn't required to examine the complainant and the by respondent Judge Nemesio S. Felix.
highlighted the fact that the receipt issued was not in witnesses.
Samianos or Sacriz name hence the proceeding in
Petitioners Vicente Lim, Sr. and Susana Lim filed with
the trial court was infirm from the onset. He shall only personally evaluate the report
the respondent court several motions and
and supporting documents submitted by the fiscal
manifestations, among others was an order be issued
regarding the existence of probable cause and issue
ISSUE: Whether or not the Court of Appeals is requiring the transmittal of the initial records of the
a warrant of arrest on the basis thereof.
correct. preliminary inquiry or investigation conducted by the
Municipal Judge Barsaga of Masbate for the best
Also, if he finds no probable cause, he may disregard
enlightenment of this Honorable Court in its personal
HELD: No. The testimonies of the two witnesses, the fiscal's report and required the submission
determination of the existence of a probable cause or
coupled with the object and documentary evidence of supporting affidavits of witnesses to aid him in
prima facie evidence as well as its determination of
they presented, are sufficient to establish the arriving at a conclusion as to the existence of
the existence of guilt, pursuant to the mandatory
existence of probable cause. From what they have probable cause.
mandate of the constitution that no warrant
witnessed, there is reason to believe that Maxicorp
shall issue unless the issuing magistrate shall
engaged in copyright infringement and unfair Otherwise, judges would be burdened with
have himself been personally convinced of
competition to the prejudice of Microsoft. Both NBI preliminary investigation instead of hearing cases.
such probable cause.
Agent Samiano and Sacriz were clear and insistent
that the counterfeit software were not only displayed
Lim vs Felix
and sold within Maxicorps premises, they were also Respondent court issued an order denying for lack of
produced, packaged and in some cases, installed merit the motions and manifestations and issued
there. FACTS: On March 17, 1989, at about 7:30 o'clock in warrants of arrest against the accused including the
the morning, at the vicinity of the airport road of the petitioners herein.
Masbate Domestic Airport, located at the
The fact that the receipt issued was not in Samianos
municipality of Masbate province of Masbate,
name nor was it in Sacriz name does not render the ISSUE : Whether or not a judge may issue a warrant
Congressman Moises Espinosa, Sr. and his security
issuance of the warrant void. No law or rule states of arrest without bail by simply relying on the
escorts, namely Provincial Guards Antonio Cortes,
that probable cause requires a specific kind of prosecution's certification and recommendation that
Gaspar Amaro, and Artemio Fuentes were attacked
evidence. No formula or fixed rule for its a probable cause exists.
and killed by a lone assassin. Dante Siblante another
determination exists. Probable cause is determined in
security escort of Congressman Espinosa, Sr.
the light of conditions obtaining in a given
survived the assassination plot, although, he himself HELD: The questioned Order of respondent Judge
situation.Thus, it was improper for the Court of
suffered a gunshot wound. An investigation of the Nemesio S. Felix of Branch 56, Regional Trial Court
Appeals to reverse the RTCs findings simply because
incident then followed. of Makati dated July 5, 1990 isdeclared NULL and
the sales receipt evidencing NBI Agent Samianos
VOID and SET ASIDE.
purchase of counterfeit goods is not in his name.
Thereafter, and for the purpose of preliminary
investigation, the designated investigator filed an RD: As held in Soliven v. Makasiar, the Judge does not
Soliven v Makasiar Nov 14, 1988 G.R. No.
amended complaint with the Municipal Trial Court of have to personally examine the complainant and his
82585
Masbate accusing Vicente Lim, Sr. et al of the crime witnesses. The Prosecutor can perform the same
of multiple murder and frustrated murder in functions as a commissioner for the taking of the
connection with the airport incident. evidence. However, there should be
(Topic on Warrant Issued by RTC) necessary documents and a report supporting the
Fiscal's bare certification.
After conducting the preliminary investigation, the
Facts:Soliven broadcasted the statement that
court issued an order concluding that a probable
President Aquino hid under her bed during a coup d'
cause has been established for the issuance of a All of these should be before the Judge.
etat. The President sued for libel. Soliven claimed
warrant of arrest of named accused..
that he can't be sued because the President was
immune from suit. We cannot determine beforehand howcursory or exh
On October 30, 1989, Fiscal Alfane filed with the austive the Judge's examination Should be.
Issue: WON Beltran's rights were violated when the Regional Trial Court of Masbate, four (4) separate Usually, this depends on the circumstances of each
RTC issued a warrant of arrest without personally informations of murder against the twelve (12) case. The Judge has to exercise sound discretion;
examining the complainant and the witnesses to accused with a recommendation of no bail. after all, the personal determination is vested in the
13 | P a g e
Copyright2015-2018 T.P. All rights reserved.
CONSTITUTIONAL LAW II : ARREST SEARCHES AND SEIZURES

3
Judge by the Constitution. However, to be sure, the City per Administrative Matter No. 87-2-244. 1990.
Judge must go beyond the Prosecutor's certification
and investigation report whenever necessary. As
Only Felipe Galarion was tried and found guilty as From the aforesaid resolution and order, petitioner
mentioned in the facts (stated above),
charged. The rest of the accused remained at large. filed a Petition for Review 4 with the Department of
the Lims presented documents of recantations of the
Felipe Galarion, however, escaped from detention Justice. Thereafter, he submitted a Supplemental
witnesses.
and has not been apprehended since then. Petition with Memorandum, 5 and then a
Supplemental Memorandum with Additional
Although, the general rule is that recantations are Exculpatory/Exonerating Evidence
In an amended information filed on October 6, 1988,
not given much weight in the determination of a case Annexed, 6 attaching thereto an affidavit of Roxas
Felizardo Roxas, alias "Ely Roxas," "Fely Roxas" and
and in the dated June 20, 1990 and purporting to be a retraction
"Lolong Roxas," was included as a co-accused. Roxas
granting of a new trial the respondent Judge before is of his affidavit of March 30, 1990 wherein he
retained petitioner Paderanga as his counsel.
suing his own warrants of arrest should, at the very implicated herein petitioner.
least, have gone over the records of the preliminary
examination conducted earlier in the As counsel for Roxas, petitioner filed, among others,
On August 10, 1990, the Department of Justice,
lightof the evidence now presented by theconcerned an Omnibus Motion to dismiss, to Quash the Warrant
through respondent Undersecretary Silvestre H. Bello
witnesses in view of the"political undertones" prevaili of Arrest and to Nullify the Arraignment on October
III, issued Resolution No. 648 7 dismissing the said
ng in the cases. 14, 1988. The trial court in an order dated January 9,
petition for review. His motion for reconsideration
1989, denied this omnibus motion but directed the
having been likewise denied, petitioner then flied the
City Prosecutor "to conduct another preliminary
In making the required personaldetermination, a Judg instant petition for mandamus and prohibition.
investigation or reinvestigation in order to grant the
e is not precludedfrom relying on the evidence ealier
accused all the opportunity to adduce whatever
gathered by responsible officers. Theextent of the reli
evidence he has in support of his defense." Issues: (1) that the preliminary investigation as to
ance depends on thecircumstances of each case and
him was not complete; and (2) that there exists
is subject to the Judge's sound discretion. However,
no prima facie evidence or probable cause to justify
the Judge abuses that discretion when having no In the course of the preliminary investigation,
his inclusion in the second amended information.
evidence before him, he issues a warrant of arrest. through a signed affidavit, Felizardo Roxas implicated
Indubitably, the respondent Judge (Felix) committed herein petitioner in the commission of the crime
a grave error when he relied solely on the charged. Ruling: Preliminary investigation is generally
Prosecutors certification and issued the questioned inquisitorial, and it is often the only means of
Order dated July 5,1990 without having before him discovering the persons who may be reasonably
The City Prosecutor of Cagayan de Oro City inhibited
any other basis for his personal determination of the charged with a crime, to enable the fiscal to prepare
himself from further conducting the preliminary
existence of a probable cause. his complaint or information. It is not a trial of the
investigation against petitioner at the instance of the
case on the merits and has no purpose except that of
latter's counsel, per his resolution dated July 7, 1989.
determining whether a crime has been committed
Panderanga vs Drilon In his first indorsement to the Department of Justice,
and whether there is probable cause to believe that
dated July 24, 1989, said city prosecutor requested
the accused is guilty thereof, and it does not place
the Department of Justice to designate a state
REGALADO, J.: p the person against whom it is taken in jeopardy.
prosecutor to continue the preliminary investigation
against herein petitioner.
In this special civil action for mandamus and A careful analysis of the circumstances obtaining in
prohibition with prayer for a writ of preliminary the present case, however, will readily show that the
In a resolution dated September 6,
injunction/restraining order, petitioner seeks to enjoin same does not fall under any of the aforesaid
1989, 1 respondent State Prosecutor Henrick F.
herein public respondents from including the former exceptions. Hence, the petition at bar must be
Gingoyon, who was designated to continue with the
as an accused in Criminal Case No. 86-39 for multiple dismissed.
conduct of the preliminary investigation against
murder, through a second amended information, and
petitioner, directed the amendment of the previously
to restrain them from prosecuting him.
amended information to include and implead herein 1. Petitioner avers that he was deprived of a full
petitioner as one of the accused therein. Petitioner preliminary investigation by reason of the fact that at
The records disclose that on October 16, 1986, an moved for reconsideration, 2 contending that the the time the resolution of September 6, 1989 was
information for multiple murder was filed in the preliminary investigation was not yet completed issued, there were still several incidents pending
Regional Trial Court, Gingoog City, against Felipe when said resolution was promulgated, and that he resolution such as the validity of the testimonies and
Galarion, Manuel Sabit, Cesar Sabit, Julito Ampo, was deprived of his right to present a corresponding affidavits of Felizardo Roxas and Rogelio Hanopol as
Eddie Torion, John Doe, Peter Doe and Richard Doe, counter-affidavit and additional evidence crucial to bases for preliminary investigation, the polygraph
for the deaths on May 1, 1984 of Renato Bucag, his the determination of his alleged "linkage" to the test of Roxas which he failed, and the clarificatory
wife Melchora Bucag, and their son Renato Bucag II. crime charged. The motion was, however, denied by questions which were supposed to be propounded by
Venue was, however, transferred to Cagayan de Oro respondent Gingoyon in his order dated January 29, petitioner's counsel to Roxas and Hanopol. Petitioner

14 | P a g e
Copyright2015-2018 T.P. All rights reserved.
CONSTITUTIONAL LAW II : ARREST SEARCHES AND SEIZURES

likwise claims that he was deprived of the 2. Petitioner further submits that there is no prima examine all other evidence submitted by the
opportunity to file his counter-affidavit to the facie evidence, or probable cause, or sufficient complainant and, where the fiscal sets a hearing to
subpoena of April 25, 1989. These contentions are justification to hold him to a tedious and prolonged propound clarificatory questions to the parties or
without merit. public trial, on the basis of the following grounds: the their witnesses, to be afforded an opportunity to be
questioned resolution of respondent Gingoyon is full present but without the right to examine or cross-
of factual misrepresentations or misapprehensions; examine. Thus, even if petitioner was not given the
Firstly, it will be noted that petitioner had already
respondent's reliance on the decision of the Regional opportunity to cross-examine Galarion and Hanopol
filed his counter-affidavit, pursuant to the subpoena
Trial Court against Felipe Galarion suffers from at the time they were presented to testify during the
issued to him on April 17, 1989, wherein he
constitutional and procedural infirmities considering separate trial of the case against Galarion and Roxas,
controverted the charge against him and dismissed it
that petitioner was not a party thereto, much less he cannot assert any legal right to cross-examine
as a malicious design of his political opponents and
was he given any opportunity to comment on or them at the preliminary investigation precisely
enemies to link him to the crime. We hold that this is
rebut the prosecution evidence; reliance on Rogelio because such right was never available to him. The
sufficient compliance with the procedural
Hanopol's testimony is likewise "contemptible," it admissibility or inadmissibility of said testimonies
requirement of the Rules of Court, specifically Section
being merely hearsay in addition to the fact that should be ventilated before the trial court during the
3(b) of Rule 112 thereof. Besides, petitioner failed to
petitioner was never given the opportunity to cross- trial proper and not in the preliminary investigation.
show that the subpoena issued on April 25, 1989
examine Hanopol at the time he testified in court;
involved a separate complaint charging an offense
and the affidavit of Roxas dated March 30, 1989,
different and distinct from that charged in the Furthermore, the technical rules on evidence are not
which is the only evidence against petitioner, has
complaint attached to the first subpoena issued to binding on the fiscal who has jurisdiction and control
been rendered nugatory by his affidavit of retraction
him earlier. over the conduct of a preliminary investigation. If by
dated June 20, 1990.
its very nature a preliminary investigation could be
waived by the accused, we find no compelling
Secondly, the veracity and credibility of the
A preliminary investigation is defined as an inquiry or justification for a strict application of the evidentiary
witnesses and their testimonies are matters of
proceeding for the purpose of determining whether rules. In addition, considering that under Section 8,
defense best addressed to the trial court for its
there is sufficient ground to engender a well founded Rule 112 of the Rules of Court, the record of the
appreciation and evaluation.
belief that a crime cognizable by the Regional Trial preliminary investigation does not form part of the
Court has been committed and that the respondent record of the case in the Regional Trial Court, then
Thirdly, the right of petitioner to ask clarificatory is probably guilty thereof, and should be held for the testimonies of Galarion and Hanopol may not be
questions is not absolute. The fiscal has the trial. 13 The quantum of evidence now required in admitted by the trial court if not presented in
discretion to determine whether or not he will preliminary investigation is such evidence sufficient evidence by the prosecuting fiscal. And, even if the
propound these questions to the parties or witnesses to "engender a well founded belief as to the fact of prosecution does present such testimonies, petitioner
concerned. As clearly provided for under Section the commission of a crime and the respondent's can always object thereto and the trial court can rule
3(e), Rule 112 of the Rules of Court.: probable guilt thereof. A preliminary investigation is on the admissibility thereof; or the petitioner can,
not the occasion for the full and exhaustive display of during the trial, petition said court to compel the
the parties' evidence; it is for the presentation of presentation of Galarion and Hanopol for purposes of
(e) If the investigating officer believes that there are
such evidence only as may engender a wen cross-examination.
matters to be clarified, he may set a hearing to
grounded belief that an offense has been committed
propound clarificatory questions to the parties or
and that the accused is probably guilty thereof. 14 We
their witnesses, during which the parties shall be WHEREFORE, the instant petition is hereby
are in accord with the state prosecutor's findings in
afforded an opportunity to be present but without the DISMISSED for lack of merit.
the case at bar that there exists prima facie evidence
right to examine or cross-examine. If the parties so
of petitioner's involvement in the commission of the
desire, they may submit questions to the to the
crime, it being sufficiently supported by the evidence PADERANGA vs DRILON
investigating officer which the latter may propound
presented and the facts obtaining therein.
to the parties or witnesses concerned.
FACTS:
Likewise devoid of cogency is petitioner's argument
Lastly, it has been held that "the proper forum before
that the testimonies of Galarion and Hanopol are
which absence of preliminary investigation should be Definition of Preliminary Examination Generally
inadmissible as to him since he was not granted the
ventilated is the Court of First Instance of a inquisitorial, often only means of discovering the
opportunity of cross-examination.
preliminary investigation does not go to the persons who may be reasonably charged with a
jurisdiction of the court but merely to the regularity crime, to enable the fiscal to prepare his complaint or
of the proceedings. It could even be waived. Indeed, It is a fundamental principle that the accused in a information.
it is frequently waived. These are matters to be preliminary investigation has no right to cross-
inquired into by the trail court not an appellate examine the witnesses which the complainant may
The institution of a criminal action depends upon the
court." 12 present. Section 3, Rule 112 of the Rules of Court
sound discretion of the Fiscal. He has the quasi-
expressly provides that the respondent shall only
judicial discretion to determine wither or not a
have the right to submit a counter-affidavit, to
15 | P a g e
Copyright2015-2018 T.P. All rights reserved.
CONSTITUTIONAL LAW II : ARREST SEARCHES AND SEIZURES

criminal case should be filed in Court. Regional Office XII against herein petitioners and six petitioners and three (3) other respondents. [8] He
(6) other persons[1] in connection with the death of a thus recommended the filing of charges against
certain Abdul Dimalen, the former COMELEC herein petitioners Bai UnggieAbdula and Odin
General Rule:
Registrar of Kabuntalan, Maguindanao.[2] The Abdula, as principals by inducement, and against the
complaint alleged that herein petitioners paid the six three (3) others, as principals by direct participation.
Injunction will not be granted to restrain a criminal other respondents the total amount of P200,000.00
prosecution for the death of Abdul Dimalen.[3]
Likewise in this 28 December 1994 Resolution,
Provincial Prosecutor Salick U. Panda, who conducted
Exception (Brocka vs Enrile): Acting on this complaint, the Provincial Prosecutor of the earlier preliminary investigation of the murder
Maguindanao, Salick U. Panda, in a Resolution dated charge, added a notation stating that he was
22 August 1994[4], dismissed the charges of murder inhibiting himself from the case and authorizing the
1. Afford adequate protection to the
against herein petitioners and five other respondents investigating prosecutor to dispose of the case
constitutional rights of the accused
on a finding that there was no prima facie case for without his approval. The reasons he cited were that
murder against them. Prosecutor Panda, however, the case was previously handled by him and that the
2. Necessary for the orderly administration of recommended the filing of an information for murder victim was the father-in-law of his son.[9]
justice or to avoid oppression or multiplicity of against one of the respondents, a certain Kasan
actions Mama. Pursuant to this Resolution, an information for
On 2 January 1995, an information for murder dated
murder was thereafter filed against Kasan Mama
28 December 1994[10] was filed against the petitioner
before the sala of respondent Judge.
3. When there is a prejudicial question spouses and Kasan Mama, Cuenco Usman and Jun
Mama before Branch 14 of the Regional Trial Court of
In an Order dated 13 September 1994[5], respondent Cotabato City, then the sala of respondent judge.
4. When the acts of the officers are without or
Judge ordered that the case, now docketed as This information was signed by investigating
excess of authority
Criminal Case No. 2332, be returned to the Provincial prosecutor Enok T. Dimaraw. A notation was likewise
Prosecutor for further investigation. In this Order, made on the information by Provincial Prosecutor
5. Double jeopardy is clearly apparent respondent judge noted that although there were Panda, which explained the reason for his inhibition.
[11]
eight (8) respondents in the murder case, the
information filed with the court "charged only one (1)
6. When the Court has no jurisdiction over the
of the eight (8) respondents in the name of Kasan
offense
Mama without the necessary resolution required
under Section 4, Rule 112 of the Revised Rules of
7. A case of persecution rather than prosecution Court to show how the investigating prosecutor The following day, or on 3 January 1995, the
arrived at such a conclusion." As such, the respondent judge issued a warrant[12] for the arrest of
respondent judge reasons, the trial court cannot petitioners. Upon learning of the issuance of the said
8. The charges are manifestly false and
issue the warrant of arrest against Kasan Mama. warrant, petitioners filed on 4 January 1995 an
motivated by vengeance
Urgent Ex-parte Motion[13] for the setting aside of the
warrant of arrest on 4 January 1995. In this motion,
Upon the return of the records of the case to the
9. Clearly no Prima Facie case against the petitioners argued that the enforcement of the
Office of the Provincial Prosecutor for Maguindanao, it
accused warrant of arrest should be held in abeyance
was assigned to 2nd Assistant Prosecutor Enok T.
considering that the information was prematurely
Dimaraw for further investigation. In addition to the
filed and that the petitioners intended to file a
The right of the accused to ask clarificatory evidence presented during the initial investigation of
petition for review with the Department of Justice.
questions is not ABSOLUTE. the murder charge, two new affidavits of witnesses
were submitted to support the charge of murder
against herein petitioners and the other respondents A petition for review[14] was filed by the petitioners
QUANTUM OF EVIDENCE required in preliminary
in the murder complaint. Thus, Prosecutor Dimaraw with the Department of Justice on 11 January 1995.
investigation is such such evidence sufficient to [15]
treated the same as a refiling of the murder charge Despite said filing, respondent judge did not act
engender a well-founded belief as to the fact of the
and pursuant to law, issued subpoena to the upon petitioners pending Motion to Set Aside the
omission of a crime and respondents probable guilt.
respondents named therein.[6] On December 6, 1994, Warrant of Arrest.
herein petitioners submitted and filed their joint
Abdula vs guiani counter-affidavits.

FACTS: On 24 June 1994, a complaint for murder, After evaluation of the evidence, Prosecutor
ISSUES:
docketed as I.S. No. 94-1361, was filed before the Dimaraw, in a Resolution dated 28 December 1994,
Criminal Investigation Service Command, ARMM [7]
found a prima facie case for murder against herein

16 | P a g e
Copyright2015-2018 T.P. All rights reserved.
CONSTITUTIONAL LAW II : ARREST SEARCHES AND SEIZURES

1. Whether the Second Information for murder Probable Cause. examination depends on the exercise of his sound
filed is valid? discretion as the circumstances of the case require.
In the case at bench, the respondent had before him
Ho vs People (Inting)
two different informations and resolutions charging
2. Validity of the Warrant of Arrest issued
two different sets of suspects. In the face of these
against petitioners.
1. DETERMINATION OF PROBABLE CAUSE BY conflicting resolutions, it behooves him not to take
THE PROSECUTOR: the certification of the investigating prosecutor at
HELD: face value. The circumstances thus require that
respondent look beyond the bare certification of the
Whether there is a reasonable ground to believe that
investigating prosecutor and examine the documents
Petition for Certiorari and Prohibition are Granted. the accused is guilty of the offense charge and
supporting the prosecutors determination of
should be held for trial is what the prosecutor passes
probable cause. The inordinate haste that attended
on.
RATIONALE: the issuance of the warrant of arrest and
respondents own admission are circumstances that
Determination of Probable Cause by a Judge: Warrant tend to belie any pretense of the fulfillment of this
In order to disqualify a Judge on the basis of
of Arrest duty.
Prejudice, petitioner must prove the same by clear
and convincing evidence.
1. The Judge should decide independently, Clearly, respondent judge, by merely stating that he
hence, he must have supporting evidence, had no reason to doubt the validity of the
Rules of Court: No complaint or information shall be
other than the Prosecutors bare report, certification made by the investigating prosecutor
filed or dismissed by an investigating Fiscal without
upon which to legally sustain his own has abdicated his duty under the Constitution to
the prior written authority or approval of the
findings on the existence of probable cause determine on his own the issue of probable cause
Provincial or City Fiscal or Chief of State Prosecutor. A
to issue an arrest order. before issuing a warrant of arrest. Consequently, the
complaint or information can only be filed if it is
warrant of arrest should be declared null and void.
approved or authorized by the Provincial or City
Fiscal or Chief of State Prosecutor. In the case at bench, respondent admits that he
issued the questioned warrant as there was "no PEOPLE VS.BENHUR MAMARIL
reason for (him) to doubt the validity of the G.R. No. 147607. January 22, 2004
Soliven vs Makasiar, In satisfying himself of the
certification made by the Assistant Prosecutor that a
existence of probable cause, the Judge is not
preliminary investigation was conducted and that Facts: SPO2 ChitoEsmenda applied before the RTC
required to personally examine the complainant and
probable cause was found to exist as against those for a search warrant authorizing the search for
his witnesses.
charged in the information filed." The statement is an marijuana at the family residence of appellant
admission that respondent relied solely and Benhur. During the search operation, the searching
Ho vs People, In the case at bench, respondent completely on the certification made by the fiscal team confiscated sachets of suspected marijuana
admits that the issued Warrant is questionable as that probable cause exists as against those charged leaves. Police officers took pictures of the confiscated
there was no reason for him to doubt the validity of in the information and issued the challenged warrant items and prepared a receipt of the property seized
the Certification made by the Assistant Prosecutor of arrest on the sole basis of the prosecutors findings and certified that the house was properly searched
that a Preliminary Investigation was conducted and and recommendations. He adopted the judgment of which was signed by the appellant and the barangay
that Probable Cause was found to exist as against the prosecutor regarding the existence of probable officials who witnessed the search.
those charged in the information filed. cause as his own. After the search, the police officers brought appellant
and the confiscated articles to the PNP station. After
weighing the specimens and testing the same, the
DOCTRINE, the Judge shall: Although the prosecutor enjoys the legal
PNP Crime Laboratory issued a report finding the
presumption of regularity in the performance of his
specimens to be positive to the test for the presence
official duties, which in turn gives his report the
1. Personally evaluate the report and the of marijuana. Moreover, the person who conducted
presumption of accuracy, nothing less than the
supporting documents submitted byt the the examination on the urine sample of appellant
fundamental law of the land commands the judge to
fiscal regarding the existence of Probable affirmed that it was positive for the same.
personally determine probable cause in the issuance
Cause and, on the basis , issue a warrant of Appellant denied that he was residing at his parents
of warrants of arrest. A judge fails in this
arrest house since he has been residing at a rented house
constitutionally mandated duty if he relies merely on
and declared that it was his brother and the latters
the certification or report of the investigating officer.
family who were residing with his mother, but on said
2. If, on the basis thereof he finds no probable
search operation, his brother and family were out. He
cause, he may disregard the fiscals report
To be sure, we cannot determine beforehand how testified that he was at his parents house because
and require the submission of supporting
cursory or exhaustive the respondents examination he visited his mother, that he saw the Receipt of
affidavits of witnesses to aid him in arriving
of the records should be.[42] The extent of the judges Property Seized for the first time during the trial and
at a conclusion as to the existence of
17 | P a g e
Copyright2015-2018 T.P. All rights reserved.
CONSTITUTIONAL LAW II : ARREST SEARCHES AND SEIZURES

admitted that the signature on the certification that mandated by the law that the examination of the that the leading questions propounded by Judge
the house was properly search was his. complaint and his witnesses must be under oath and Lorenzo on Tan does not detract from the fact that
reduced to writing in the form of searching questions searching questions were also propounded on the
Issues: 1) Whether or not the trial court erred in and answers was not complied with, rendering the witnesses, and that based on the entirety of such
issuing a search warrant. search warrant invalid. Consequently, the evidence propounded questions and the latters answers, there
seized pursuant to illegal search warrant cannot be was probable cause for the issuance of a search
2) Whether or not the accused-appellant waived his used in evidence against appellant in accordance warrant.
right to question the legality of the search. with Section 3 (2) Article III of the Constitution.
ISSUE: WON the search warrant issued by the judge
3) Whether or not evidence seized pursuant to an
People vs De Los Reyes G.R. 140657 is valid.
illegal search be used as evidence against the
accused.
FACTS: On June 18, 1998, SPO3 Benjamin Nuguid of HELD: The mandate of the Judge is for him to
Held: 1) The issuance of a search warrant is justified the Western Police District applied for a search conduct a full and searching examination of the
only upon a finding of probable cause. Probable warrant with the RTC of Manila, Branch 43, against complainant and the witnesses he may produce. In
cause for a search has been defined as such facts Cesar Reyes alias "Cesar Itlog." In support of his the absence of a rule to the contrary, the
and circumstances which would lead a reasonably application, Nuguid submitted his affidavit and that determination of probable cause cannot be delegated
discreet and prudent man to believe that an offense of Alexis Tan, a housewife. Nuguid and Tan also by the Judge, in part, or in whole, regardless of the
has been committed and that the objects sought in testified in support of the application. After the court qualifications of the person on whom reliance is
connection with the offense are in the place sought conducted examination of the said witnesses, it placed. It is not permissible for the Judge to share the
to be searched. In determining the existence of issued on even date Search Warrant No. 98-905 required determination with another.
probable cause, it is required that: 1) The judge must authorizing the search of the house allegedly under
examine the complaint and his witnesses personally; the possession and custody of one Cesar Reyes alias
The searching questions propounded to the applicant
2) the examination must be under oath; 3) the "Cesar Itlog," at No. 2600 Oroquieta Street, Sta. Cruz,
and the witnesses must depend on a large extent
examination must be reduced in writing in the form Manila.
upon the discretion of the Judge. Although there is no
of searching questions and answers. The prosecution
hard-and-fast rule as to how a Judge may conduct his
failed to prove that the judge who issued the warrant
The policemen conducted a search not only of the examination, it is axiomatic that the said
put into writing his examination of the applicant and
house at No. 2600 Oroquieta Street, Sta Cruz, Manila, examination must be probing and exhaustive and not
his witnesses on the form of searching questions and
which turned out to be the house of respondent merely routinary, general, peripheral or perfunctory.
answers before issuance of the search warrant. Mere
Cesar delos Reyes, but also of the car and motorcycle
affidavits of the complainant and his witnesses are
owned by De Los Reyes. They found several firearms,
not sufficient. Such written examination is necessary The questions propounded on Nuguid by Judge
38 pcs of valium, 18 transparent plastic bags with
in order that the judge may be able to properly Lorenzo were not searching and probing, but merely
total net weight of 886.8 grams of white crystalline
determine the existence and non-existence of superficial and perfunctory.
substance known as "shabu".
probable cause. Therefore, the search warrant is
tainted with illegality by failure of the judge to
Even a cursory reading of the transcript will show
conform with the essential requisites of taking the The respondent filed a motion to quash the
that most of the questions propounded on Tan by the
examination in writing and attaching to the record, informations but was denied by the trial court
Judge were leading questions, and that those which
rendering the search warrant invalid.
were not leading were merely based on or related to
2) At that time the police officers presented the
The respondent then filed a petition for certiorari in the answers earlier given to the leading questions.
search warrant, appellant could not determine if the
the CA, alleging that the questions propounded by By asking such leading questions, the Judge thereby
search warrant was issued in accordance with law. It
RTC Judge Manuela F. Lorenzo on Nuguid and Tan supplied the answers to her questions.
was only during the trial that appellant, through his
were leading and not searching. He also alleged that
counsel, had reason to believe that the search
Judge Lorenzo delegated the examination of Tan to
warrant was illegally issued. Moreover, appellant Thus, in issuing a search warrant, the Judge must
Nuguid, and allowed the latter to question her.
seasonably objected on constitutional grounds to the strictly comply with the requirements of the
admissibility of the evidence seized pursuant to said Constitution and the statutory provisions. A liberal
warrant during the trial, after the prosecution The CA rendered a Decision granting the petition and construction should be given in favor of the
formally offered its evidence. Under the nullifying the search warrant and held that the RTC individual to prevent stealthy encroachment upon, or
circumstances, no intent to waive his rights can delegated its duty to determine probable cause to gradual depreciation of the rights secured by the
reasonably be inferred from his conduct before or the applicant; the application for a search warrant Constitution. No presumption of regularity is to be
during the trial. was based on hearsay evidence; invoked in aid of the process when an officer
3) No matter how incriminating the articles taken undertakes to justify it. PETITION IS DENIED.
from the appellant may be, their seizure cannot
The People of the Philippines filed the instant petition
validate an invalid warrant. The requirement
for review of the decision. The petitioner also asserts Pasion Vda. De Garcia vs. Locsin G.R. No. L-
18 | P a g e
Copyright2015-2018 T.P. All rights reserved.
CONSTITUTIONAL LAW II : ARREST SEARCHES AND SEIZURES

45950 Almeda. He did not decide for himself. It does The CA rendered a Decision granting the petition and
not appear that he examined the applicant and his nullifying the search warrant and held that the RTC
witnesses, if any. Even accepting the description of delegated its duty to determine probable cause to
FACTS: Mariano G. Almeda, an agent of the Anti-
the properties to be seized to be sufficient and on the the applicant; the application for a search warrant
Usuary Board, obtained from the justice of the peace
assumption that the receipt issued is sufficiently was based on hearsay evidence;
of Tarlac, a search warrant commanding any officer
detailed within the meaning of the law, the
of the law to search the person, house or store of the
properties seized were not delivered to the court
petitioner at Victoria, Tarlac, for certain books, lists, The People of the Philippines filed the instant petition
which issued the warrant, as required by law.
chits, receipts, documents and other papers relating for review of the decision. The petitioner also asserts
to her activities as usurer. The search warrant was that the leading questions propounded by Judge
issued upon an affidavit given by the said Almeda. Instead, they were turned over to the resp. provincial Lorenzo on Tan does not detract from the fact that
fiscal & used by him in building up cases against searching questions were also propounded on the
petitioner. Considering that at the time the warrant witnesses, and that based on the entirety of such
On the same date, the said Mariano G. Almeda,
was issued, there was no case pending against the propounded questions and the latters answers, there
accompanied by a captain of the Philippine
petitioner, the averment that the warrant was issued was probable cause for the issuance of a search
Constabulary, went to the office of the petitioner in
primarily for exploration purposes is not without warrant.
Victoria, Tarlac and, after showing the search warrant
basis.
to the petitioners bookkeeper, Alfredo Salas, and,
without the presence of the petitioner who was ill ISSUE: WON the search warrant issued by the judge
and confined at the time, proceeded with the People vs De Los Reyes G.R. 140657 is valid.
execution thereof
FACTS: On June 18, 1998, SPO3 Benjamin Nuguid of HELD: The mandate of the Judge is for him to
The papers and documents seized were kept for a the Western Police District applied for a search conduct a full and searching examination of the
considerable length of time by the Anti-Usury Board warrant with the RTC of Manila, Branch 43, against complainant and the witnesses he may produce. In
and thereafter were turned over by it to the Cesar Reyes alias "Cesar Itlog." In support of his the absence of a rule to the contrary, the
respondent fiscal who subsequently filed six separate application, Nuguid submitted his affidavit and that determination of probable cause cannot be delegated
criminal cases against the herein petitioner for of Alexis Tan, a housewife. Nuguid and Tan also by the Judge, in part, or in whole, regardless of the
violation of the Anti-Usury Law. testified in support of the application. After the court qualifications of the person on whom reliance is
conducted examination of the said witnesses, it placed. It is not permissible for the Judge to share the
issued on even date Search Warrant No. 98-905 required determination with another.
The legality of the search warrant was challenged by
authorizing the search of the house allegedly under
counsel for the petitioner in the six criminal cases
the possession and custody of one Cesar Reyes alias
and the devolution of the documents demanded. The The searching questions propounded to the applicant
"Cesar Itlog," at No. 2600 Oroquieta Street, Sta. Cruz,
respondent Judge denied the petitioners motion for and the witnesses must depend on a large extent
Manila.
the reason that though the search warrant was upon the discretion of the Judge. Although there is no
illegal, there was a waiver on the part of the hard-and-fast rule as to how a Judge may conduct his
petitioner. The policemen conducted a search not only of the examination, it is axiomatic that the said
house at No. 2600 Oroquieta Street, Sta Cruz, Manila, examination must be probing and exhaustive and not
which turned out to be the house of respondent merely routinary, general, peripheral or perfunctory.
HELD: Freedom from unreasonable searches and
Cesar delos Reyes, but also of the car and motorcycle
seizures is declared a popular right and for a search
owned by De Los Reyes. They found several firearms,
warrant to be valid, (1) it must be issued upon The questions propounded on Nuguid by Judge
38 pcs of valium, 18 transparent plastic bags with
probable cause; (2) the probable cause must be Lorenzo were not searching and probing, but merely
total net weight of 886.8 grams of white crystalline
determined by the judge himself and not by superficial and perfunctory.
substance known as "shabu".
the applicant or any other person; (3) in the
determination of probable cause, the judge must
Even a cursory reading of the transcript will show
examine, under oath or affirmation, the complainant The respondent filed a motion to quash the
that most of the questions propounded on Tan by the
and such witnesses as the latter may produce; and informations but was denied by the trial court
Judge were leading questions, and that those which
(4) the warrant issued must particularly describe the
were not leading were merely based on or related to
place to be searched and persons or things to be
The respondent then filed a petition for certiorari in the answers earlier given to the leading questions.
seized.
the CA, alleging that the questions propounded by By asking such leading questions, the Judge thereby
RTC Judge Manuela F. Lorenzo on Nuguid and Tan supplied the answers to her questions.
In the instant case the existence of probable were leading and not searching. He also alleged that
cause was determined not by the judge himself Judge Lorenzo delegated the examination of Tan to
Thus, in issuing a search warrant, the Judge must
but by the applicant. All that the judge did was Nuguid, and allowed the latter to question her.
strictly comply with the requirements of the
to accept as true the affidavit made by agent
Constitution and the statutory provisions. A liberal
19 | P a g e
Copyright2015-2018 T.P. All rights reserved.
CONSTITUTIONAL LAW II : ARREST SEARCHES AND SEIZURES

construction should be given in favor of the execution thereof (4) the warrant issued must particularly describe the
individual to prevent stealthy encroachment upon, or place to be searched and persons or things to be
gradual depreciation of the rights secured by the seized.
The papers and documents seized were kept for a
Constitution. No presumption of regularity is to be
considerable length of time by the Anti-Usury Board
invoked in aid of the process when an officer
and thereafter were turned over by it to the In the instant case the existence of probable cause
undertakes to justify it. PETITION IS DENIED.
respondent fiscal who subsequently filed six separate was determined not by the judge himself but by the
criminal cases against the herein petitioner for applicant. All that the judge did was to accept as true
Pasion Vda. De Garcia vs. Locsin G.R. No. L- violation of the Anti-Usury Law. the affidavit made by agent Almeda. He did not
45950 decide for himself. It does not appear that he
examined the applicant and his witnesses, if any.
The legality of the search warrant was challenged by
Even accepting the description of the properties to
FACTS: Mariano G. Almeda, an agent of the Anti- counsel for the petitioner in the six criminal cases
be seized to be sufficient and on the assumption that
Usuary Board, obtained from the justice of the peace and the devolution of the documents demanded. The
the receipt issued is sufficiently detailed within the
of Tarlac, a search warrant commanding any officer respondent Judge denied the petitioners motion for
meaning of the law, the properties seized were not
of the law to search the person, house or store of the the reason that though the search warrant was
delivered to the court which issued the warrant, as
petitioner at Victoria, Tarlac, for certain books, lists, illegal, there was a waiver on the part of the
required by law.
chits, receipts, documents and other papers relating petitioner.
to her activities as usurer. The search warrant was
issued upon an affidavit given by the said Almeda. Instead, they were turned over to the resp. provincial
HELD: Freedom from unreasonable searches and
fiscal & used by him in building up cases against
seizures is declared a popular right and for a search
petitioner. Considering that at the time the warrant
On the same date, the said Mariano G. Almeda, warrant to be valid, (1) it must be issued upon
was issued, there was no case pending against the
accompanied by a captain of the Philippine probable cause; (2) the probable cause must be
petitioner, the averment that the warrant was issued
Constabulary, went to the office of the petitioner in determined by the judge himself and not by the
primarily for exploration purposes is not without
Victoria, Tarlac and, after showing the search warrant applicant or any other person; (3) in the
basis.
to the petitioners bookkeeper, Alfredo Salas, and, determination of probable cause, the judge must
without the presence of the petitioner who was ill examine, under oath or affirmation, the complainant
and confined at the time, proceeded with the and such witnesses as the latter may produce; and

20 | P a g e
Copyright2015-2018 T.P. All rights reserved.

You might also like